Spring with Equal Forces

Taken from the 2011 F = MA Exam (USAPHO Qualifiers)

A spring has an equilibrium length of 2.0 meters and a spring constant of 10 newtons/meter. Alice is pulling on one end of the spring with a force of 3.0 newtons. Bob is pulling on the opposite end of the spring with a force of 3.0 newtons, in the opposite direction. What is the resulting length of the spring?

#Physics #CMC

Note by Jess Smith
7 years, 6 months ago

No vote yet
2 votes

  Easy Math Editor

This discussion board is a place to discuss our Daily Challenges and the math and science related to those challenges. Explanations are more than just a solution — they should explain the steps and thinking strategies that you used to obtain the solution. Comments should further the discussion of math and science.

When posting on Brilliant:

  • Use the emojis to react to an explanation, whether you're congratulating a job well done , or just really confused .
  • Ask specific questions about the challenge or the steps in somebody's explanation. Well-posed questions can add a lot to the discussion, but posting "I don't understand!" doesn't help anyone.
  • Try to contribute something new to the discussion, whether it is an extension, generalization or other idea related to the challenge.
  • Stay on topic — we're all here to learn more about math and science, not to hear about your favorite get-rich-quick scheme or current world events.

MarkdownAppears as
*italics* or _italics_ italics
**bold** or __bold__ bold

- bulleted
- list

  • bulleted
  • list

1. numbered
2. list

  1. numbered
  2. list
Note: you must add a full line of space before and after lists for them to show up correctly
paragraph 1

paragraph 2

paragraph 1

paragraph 2

[example link](https://brilliant.org)example link
> This is a quote
This is a quote
    # I indented these lines
    # 4 spaces, and now they show
    # up as a code block.

    print "hello world"
# I indented these lines
# 4 spaces, and now they show
# up as a code block.

print "hello world"
MathAppears as
Remember to wrap math in \( ... \) or \[ ... \] to ensure proper formatting.
2 \times 3 2×3 2 \times 3
2^{34} 234 2^{34}
a_{i-1} ai1 a_{i-1}
\frac{2}{3} 23 \frac{2}{3}
\sqrt{2} 2 \sqrt{2}
\sum_{i=1}^3 i=13 \sum_{i=1}^3
\sin \theta sinθ \sin \theta
\boxed{123} 123 \boxed{123}

Comments

By the way, please do not use the CMC tag in any new discussions. That's specifically for Cody's Math Contest.

Michael Tang - 7 years, 6 months ago

Log in to reply

Sorry, I accidentally included it. Thanks for letting me know.

Jess Smith - 7 years, 6 months ago

Thanks, you beat me to it.

Cody Johnson - 7 years, 6 months ago

First Law of Newton: If there's a force on something, it'll accelerate. Since this spring isn't moving, there's no force on it. If there wasn't any Bob pulling from the other end, Alice will drag the spring with her all the way home (and if has no mass, it'll be totally upstretched too). Picture Sally hooking one end to her bike and driving home. It won't be stretched at all.

Now how do springs behave? If you want to elongate a spring (with constant 10 N/m) by one meter, you'll have to hold both ends and pull, and keep pulling till it's 1 m longer and hold it that way. At that point, you'll be exerting 10 N force with each hand. The forces on the hands have to equal, otherwise, the spring will accelerate. Or you could tie one end to a wall and pull. Or in this case, a Bob guy who'll halfway for you. The advantage you get if you have Bob is that while stretching, he'll do half the work for you. However, once done, both exert exactly the same force; they have to. Bob is no longer necessary. A wall will do just as well.

For this problem, the constant is 10 N/m => 0.1 m/N. Since the force on the ends is 3 newtons, the elongations is 0.3 m, from which we conclude that the final length is 2.3 m.

Aditya Js - 7 years, 6 months ago

Log in to reply

OH, okay. Thanks! So what would happen if Alice was pulling with 4 N and Bob was pulling with 3 N? The spring would obviously accelerate, but how much would it stretch by?

Jess Smith - 7 years, 6 months ago

Log in to reply

Visualise this: a train engine hooked to a compartment and pulling it forward. They're connected to each other by those massive springs. Now, will the force on both ends of the spring be the same? By the way, the spring is much lighter compared to the engine and compartment.

Aditya Js - 7 years, 6 months ago

Log in to reply

@Aditya Js I don't think so; they're accelerating at the same rate, and the engine is much more massive than the compartment.

Jess Smith - 7 years, 6 months ago

Log in to reply

@Jess Smith Yeah, I used to think that too. I used to get so confused with Newton's laws. We'd think that since the engine is pulling the compartment, there should be more force on the side of the compartment. Totally understandable.

But now, let's look at it this way. If the train is accelerating with a, then the spring should have the same acceleration. So the spring has an acceleration a. Now, forget the train exists. There is black everywhere. Just plain scary empty void. And in the middle is the spring. See nothing else. The force on it should be mass times acceleration. And since its mass is negligible, so is the force.

So does that mean the spring has nearly zero force on it? Well, yes. Since, there are two forces on it, one forward and one backward, they should almost perfectly cancel. My point here is, when you are applying second law, the only mass to be taken into account is the spring, don't bring the train into the equation. It doesn't matter if it's there or not. The fact that forces on both ends is nearly equal is the next part. Before that, we deduce that force is zero. Then, since there are two forces, we say that they are nearly equal?

If you sketch out the free-body diagrams and solve the equations, you'll get the result. But try to see why this is so counter-intuitive. It might help a lot more in convincing you than all the mathematics in the world.

Aditya Js - 7 years, 6 months ago

Log in to reply

@Aditya Js So, if I'm understanding correctly, let's say Alice is pulling with a force of magnitude A and Bob is pulling with a force of magnitude B. Suppose that A > B. The force that affects the acceleration of the spring is A - B. The force that determines how much the spring stretches out is B.

Correct?

Jess Smith - 7 years, 6 months ago

Log in to reply

@Jess Smith Yes.

For the train problem though, the difference can't be too large. Because if it was so, the spring would just shoot ahead while the engine and the compartment slowly go forward, which is as ridiculous to visualise as it is impossible.

For the present problem, it is precisely zero since forces can't be applied on massless objects.

Aditya Js - 7 years, 6 months ago

Hopefully this is right: the total force on the spring is 66 Newtons, so by Hooke's law we have F=kdF = kd or 6 N=10Nmd6 \text{ N} = 10 \textstyle\frac{\text{N}}{\text{m}} \cdot d where dd is the additional length of the spring. Thus, d=0.6m,d = 0.6 \text{m}, and the spring stretches to 2.6m.\boxed{2.6 \text{m}}.

Michael Tang - 7 years, 6 months ago

Log in to reply

That's what I thought too. However, the answer key says it's 2.3m and, unfortunately, there are no solutions.

Jess Smith - 7 years, 6 months ago

kx = 6 , ................where k and x are spring constant and extension in spring.

x = 0.6

So, resultant length = 2.6m

SHUBHAM KUMAR - 7 years, 6 months ago

Log in to reply

This was my answer too, but, like I said below, the answer is actually 2.3 meters.

Jess Smith - 7 years, 6 months ago

Log in to reply

Are you sure about the authenticity of the key???

SHUBHAM KUMAR - 7 years, 6 months ago

Log in to reply

@Shubham Kumar Well, this is directly from the U.S. Physics Olympiad website, and there were no disputes listed. Here's the link in case you wanna check for yourself (2011 exam): http://www.aapt.org/physicsteam/2014/exams.cfm

Jess Smith - 7 years, 6 months ago

The spring can be thought of as two springs in series where the net spring constant is 10 N/m.

1/knet = 1/k1 + 1/k2

Since k1 = k2,

knet = 20 N/m

Therefore the extension of each spring is

3 N = 20 N/m * x

x = 0.15 m

The total extension is then 0.3 m, resulting to a net length of 2.3 m.

Not sure about this though.

Carlo Solibet - 7 years, 6 months ago

Log in to reply

Are you sure springs can be divided up in that way? I guess my main question is why don't they have the same spring coefficient.

Jess Smith - 7 years, 6 months ago

The equation you have used is for springs in series and not them in parallel..Also, in the given situation, the two portions are in series configuration.

SAMARTH M.O. - 7 years, 6 months ago

Log in to reply

I edited my post. Sorry for the typo. I meant that the springs are in series.

Carlo Solibet - 7 years, 6 months ago

Nevermind, I read up on springs in parallel and in series. Thanks!

Jess Smith - 7 years, 6 months ago

ans : 3m

Explanation : When you pull the spring on the two ends, the centre of the spring remains stationary. Now the whole sring acts like two springs in series, each with constants 2k.( in series 1/(k1) + 1/(k2) = 1/k) .. Thus for Bob and Alice, now, its like pulling springs of force constant 2k= 23 = 6 N/m.

so, 3 = 6 * x

                      x = 0.5 m

Totally, the extension is 0.5 + 0.5 = 1 m.

Thus the new length is 2 + 1 = 3 m.

SAMARTH M.O. - 7 years, 6 months ago

Log in to reply

Well, first off, the answer is actually 2.3 meters. Also, I don't think springs work like that; that is, I don't think you can just divide the spring into two smaller springs with equal spring coefficients.

Jess Smith - 7 years, 6 months ago

Log in to reply

Yes...it is meaningful to view the system as two springs in series. And, I am not dividing the spring into two with equal coefficients. The two springs are in series and their effective constant is k. So, by the equation , the new coefficient is 2k.

The mistake I did was to wrongly take in the value of k from the question as 3N/m instead of 10N/m.(you can see that in my comment above)

So, the corrected equation is :

3 = (2*10) * x

x = 0.15 m

Total extension = 0.3 m

Thus new length = 2.3 m.

SAMARTH M.O. - 7 years, 6 months ago
×

Problem Loading...

Note Loading...

Set Loading...